Doubts.

1) If \(a_1,a_2,a_3,a_4,........a_n\) are distinct odd natural numbers such that they aren't divisible by no prime number greater than 5, then maximum value of \(\sum_{i=1}^n \frac{1}{a_i}\) is

2) Solve: 2logx2^{logx} + 2logx2\sqrt2^{logx^{2}} =8

3) The sum of all integral values of a ϵ[9,9]\epsilon [-9,9] so that the equation (x2)log13(3x7a)(x-2)log_{13}(3^{x}-7a) = log132log_{13}2 + 2log13a2log_{13}a has integral solutions.

4) Find difference in the number of integer values of y and x in solution of system of inequalities: log2x(2y)>0log_{2-x}(2-y)>0 and log4y(2x2)>0log_{4-y}(2x-2)>0 .

5) Solve: 8.27log6x 8.27^{log_6x} + 27.8log6x27.8^{log_6x} - x3x>0x^{3}-x>0

Any help would be greatly appreciated.

Note by Baibhab Chakraborty
3 months ago

No vote yet
1 vote

  Easy Math Editor

This discussion board is a place to discuss our Daily Challenges and the math and science related to those challenges. Explanations are more than just a solution — they should explain the steps and thinking strategies that you used to obtain the solution. Comments should further the discussion of math and science.

When posting on Brilliant:

  • Use the emojis to react to an explanation, whether you're congratulating a job well done , or just really confused .
  • Ask specific questions about the challenge or the steps in somebody's explanation. Well-posed questions can add a lot to the discussion, but posting "I don't understand!" doesn't help anyone.
  • Try to contribute something new to the discussion, whether it is an extension, generalization or other idea related to the challenge.
  • Stay on topic — we're all here to learn more about math and science, not to hear about your favorite get-rich-quick scheme or current world events.

MarkdownAppears as
*italics* or _italics_ italics
**bold** or __bold__ bold

- bulleted
- list

  • bulleted
  • list

1. numbered
2. list

  1. numbered
  2. list
Note: you must add a full line of space before and after lists for them to show up correctly
paragraph 1

paragraph 2

paragraph 1

paragraph 2

[example link](https://brilliant.org)example link
> This is a quote
This is a quote
    # I indented these lines
    # 4 spaces, and now they show
    # up as a code block.

    print "hello world"
# I indented these lines
# 4 spaces, and now they show
# up as a code block.

print "hello world"
MathAppears as
Remember to wrap math in \( ... \) or \[ ... \] to ensure proper formatting.
2 \times 3 2×3 2 \times 3
2^{34} 234 2^{34}
a_{i-1} ai1 a_{i-1}
\frac{2}{3} 23 \frac{2}{3}
\sqrt{2} 2 \sqrt{2}
\sum_{i=1}^3 i=13 \sum_{i=1}^3
\sin \theta sinθ \sin \theta
\boxed{123} 123 \boxed{123}

Comments

Answer to second question -

8=4+4=22+24=22+222logx=28 = 4 + 4 = 2^{2} + \sqrt{2}^{4} = 2^{2} + \sqrt{2}^{2^{2}} \Rightarrow \log x = 2 x=100 \therefore \boxed{x = 100}

I will try to solve the rest and send later @Baibhab Chakraborty :)

Log in to reply

Thank you sir.

Baibhab Chakraborty - 3 months ago

Log in to reply

He is of your age too, I don't think he will like to be called sir either( but if Percy wants to be addressed so, then so be it)

Jason Gomez - 3 months ago

Log in to reply

@Jason Gomez No please don't call me sir, Percy is just fine @Baibhab Chakraborty

@Jason Gomez - I do not want to be addressed like that...don't give people ideas lmao

Log in to reply

@A Former Brilliant Member Hmm okay sir

Jason Gomez - 3 months ago

Log in to reply

@Jason Gomez Don't call me sir, दादी (google translate copy/paste lmao)

Log in to reply

@A Former Brilliant Member Okay गंदा पानी

Jason Gomez - 3 months ago

Log in to reply

@Jason Gomez Dirty water? lmao

Log in to reply

@A Former Brilliant Member I wanted it to rhyme(but ofc you wouldn’t know it rhymed)

Jason Gomez - 3 months ago

For the first one use a1=3,a2=5,a3=32,a4=3×5,a5=52,a6=33a_1=3,a_2=5,a_3=3^2,a_4=3 \times 5,a_5=5^2, a_6 = 3^3 till where required

Jason Gomez - 3 months ago

Log in to reply

Or can n tend to infinity?

Jason Gomez - 3 months ago

Log in to reply

n is a variable, I guess it might not need the value of n whatsoever.

Baibhab Chakraborty - 3 months ago

Sir, can you please explain why? Thank you sir for paying heed to my doubt.

Baibhab Chakraborty - 3 months ago

Log in to reply

Please don't call me sir, I am just 17 years old :)

Jason Gomez - 3 months ago

You don't want the numbers to be even, so no factors of two are allowed, at the same time you want the only prime factors to be less than or equal to 5, so just take the prime factors as 3 and 5 and their powers and multiples as the numbers

Jason Gomez - 3 months ago

If it can the answer is just (13+132...)(15+152...)=18(\frac13+ \frac{1}{3^2} ...) (\frac15+ \frac{1}{5^2} ...)=\frac18

Jason Gomez - 3 months ago

Log in to reply

Shouldnt it start from 1 because 1 makes the sum greater as then 3/2.5/4=15/8 . This might be the maximum value.

Baibhab Chakraborty - 3 months ago

Log in to reply

@Baibhab Chakraborty Oh yeah add an one too

Jason Gomez - 3 months ago

Yes, its just the product of two Geometric series' I think.

(1+13+132+133)(1+15+152+153)=1.5×1.25=1.875(1 + \dfrac{1}{3} + \dfrac{1}{3^{2}} + \dfrac{1}{3^{3}} \ldots)(1 + \dfrac{1}{5} + \dfrac{1}{5^{2}} + \dfrac{1}{5^{3}} \ldots) = 1.5 \times 1.25 = 1.875

Log in to reply

@A Former Brilliant Member Yes Baibhab had corrected me and written the same too…

Jason Gomez - 3 months ago

Log in to reply

@Jason Gomez Yeah, just saw that.

For the fourth one, the both inequalities have infinite solutions, so you can’t compare both

Jason Gomez - 3 months ago

I don’t know how to solve the third one, but “a” has to be greater than zero for the right hand side to be well defined

Jason Gomez - 3 months ago

@Baibhab Chakraborty - Is the fifth one 8 x 27 or 8.27 because if it is multiplication instead of decimal, I think I can solve it.

Log in to reply

I think it’s multiplication(decimal will be just too bad)

Jason Gomez - 3 months ago

Log in to reply

Yea, it will be very messy. 8 x 27 should work well because the power log is at base 6, and 8 x 27 is just 6^3

Log in to reply

@A Former Brilliant Member But I think the log is only on one of them, else it becomes too easy to solve

Jason Gomez - 3 months ago

multiplication

Baibhab Chakraborty - 3 months ago

Log in to reply

Okay. Use \times in your latex to get the multiplication cross symbol :)

Answer to fifth question, if multiplication replaces the decimals -

Let log6x=y\log_{6} x = y, then 6y=x6^{y} = x

8×27y+27×8y63y>x216(27y1+8y1216y1)>x8 \times 27^{y} + 27 \times 8^{y} - 6^{3y} > x \Rightarrow 216(27^{y-1} + 8^{y-1} - 216^{y-1}) > x

If yy is higher than 1, (27y1+8y1216y1)(27^{y-1} + 8^{y-1} - 216^{y-1}) will give us a negative value, and xx will become negative. xx cannot be negative, as 6y=x6^{y} = x.

So y1y \leq 1, and x<216(27y1+8y1216y1)x < 216(27^{y-1} + 8^{y-1} - 216^{y-1})

I don't know what exactly the question is looking for, but my best guess is a maximum value, since we have lesser than and lesser than equal to, so in that case, maximum value of x=6x = 6, because maximum value of y=1y = 1, and 6y=x6^{y} = x.

@Baibhab Chakraborty - Is this what you were looking for?

Log in to reply

I think you mean 8×27y+27×8y63y>x8 \times 27^y + 27 \times 8^y - 6^{3y} > x

Jason Gomez - 3 months ago

Log in to reply

You got every term on the left hand side wrong, congrats!!

Jason Gomez - 3 months ago

Log in to reply

@Jason Gomez Bruh, I was still editing, just went AFk for a few moments

@Jason Gomez Now it is complete, I think...lmao

According to desmos the value of x lies between 0 and 9.533 (all values in between satisfy)

Jason Gomez - 3 months ago

Log in to reply

How did you get that? What did you input?

Log in to reply

@A Former Brilliant Member The exact same equation given in the beginning

Jason Gomez - 3 months ago

Log in to reply

@Jason Gomez Wolfram gives this when I input the inequality -

1.21644×109<x<9.533021.21644 \times 10^{-9} < x < 9.53302

Log in to reply

@A Former Brilliant Member x=0 satisfies the equation, so wolfram is wrong(not enough computational power?)

Jason Gomez - 3 months ago

Log in to reply

@Jason Gomez If you use x=0 in the equation, then log x just approaches negative infinity, right? So the lhs would then just approach zero? I'm confused...

@Jason Gomez I probably did something wrong then...

nvm, I input the 3 inequalities and got 0 to 9.533 as well

exactly sir. Thank you.

Baibhab Chakraborty - 3 months ago

Log in to reply

Uh...can you, like not call me sir? Its embarrassing, I'm the same age as you...

Log in to reply

@A Former Brilliant Member @Dr.Perseus no probs at all doc

Jason Gomez - 2 months, 4 weeks ago

Log in to reply

@Jason Gomez I shall dub thee...the brick boy...

A Former Brilliant Member - 2 months, 4 weeks ago

@Baibhab Chakraborty - Are you really 16??

Log in to reply

Ah yes, I turned 16 this January only. In fact, I joined Brilliant this February. Thank you sir for your kind help.

Baibhab Chakraborty - 3 months ago

Log in to reply

Cool, I'm 16 too, so can you please stop calling me sir?

Log in to reply

Log in to reply

@Baibhab Chakraborty thank you lmao

@Siddharth Chakravarty - Help, problem 5

@Siddharth Chakravarty - Do you play Animal Jam?

A Former Brilliant Member - 2 months, 3 weeks ago

Log in to reply

Yes.

Siddharth Chakravarty - 2 months, 3 weeks ago

Log in to reply

Okay. So there's this Animal Jam Tuber named Siddharth Chakravarty, who also happens to have commented on Mahdi's yt video. Coincidence, I think not. If you really are that person, all I can say is, your voice doesn't sound right for a 16 yr old...lmao

A Former Brilliant Member - 2 months, 3 weeks ago

Log in to reply

@A Former Brilliant Member Lol that's me only but the videos are too old, liks 2 years ago and also my editor used to make mine voice a bit effiminate. XD

Siddharth Chakravarty - 2 months, 3 weeks ago

Log in to reply

@Siddharth Chakravarty you had a editor? At just 200 something subs? lmao

A Former Brilliant Member - 2 months, 3 weeks ago

Log in to reply

@A Former Brilliant Member Editor=Editing software I meant. 😂

Siddharth Chakravarty - 2 months, 3 weeks ago

Log in to reply

@Siddharth Chakravarty Oh lol ok

A Former Brilliant Member - 2 months, 3 weeks ago

Log in to reply

@A Former Brilliant Member Lol I just watched my videos with the voice and it felt so cringy 😂 BTW count me out of Mathathon as I just remembered about it today suddenly and saw now that you posted a lot of problems.

Siddharth Chakravarty - 2 months, 3 weeks ago

Log in to reply

@Siddharth Chakravarty lol okay

A Former Brilliant Member - 2 months, 3 weeks ago

@A Former Brilliant Member Lol

Baibhab Chakraborty - 2 months, 3 weeks ago

@Percy Jackson @Siddharth Chakravarty-can you please check my feed and help me on my latest doubt....

Baibhab Chakraborty - 2 months, 2 weeks ago
×

Problem Loading...

Note Loading...

Set Loading...